1
$\begingroup$

I have a doubt regarding the well-known concepts of weak Pareto optimality and monotonicity.

Let $N$ be a finite set of players, let $A$ be a finite set of alternatives, let $\mathcal{P}$ be the set of all linear order profiles on $A$, and let $F:\mathcal{P}\to 2^A\backslash\{\emptyset\}$ be a social choice correspondence.

Let $F^*:\mathcal{P}\to 2^A\backslash\{\emptyset\}$ be the weak Pareto correspondence: namely, for all linear order profiles $P\in\mathcal{P}$, \begin{gather} F^*(P)=\{x\in A\mid(\nexists y\in A)[(\forall i\in N)(yP_ix)]\} \end{gather}

Given any player $i\in N$, any alternative $x\in A$ and any linear order profile $P\in\mathcal{P}$, let $L_i(x,P)=\{y\in A\mid xP_iy\}$ be player $i$'s lower contour set at $x$.

A social choice rule $F:\mathcal{P}^N\to 2^A\backslash\{\emptyset\}$ is monotonic if and only if for all alternatives $x\in A$ and all linear order profiles $P,P'\in\mathcal{P}$, the following is true: if $x\in F(P)$ and $L_i(x,P)\subseteq L_i(x,P')$ for all $i\in N$, then $x\in F(P')$.

We know by Maskin & Sjöström (Footnote 15, p. 248, 2002) that the weak Pareto correspondence is monotonic at the unrestricted domain of linear orders.

What I am wondering is whether all subcorrespondences of the weak Pareto correspondence is also monotonic at the unrestricted domain of linear orders.

$\endgroup$

1 Answer 1

2
$\begingroup$

The question in the title seems to differ from the question in the body: all/any. At least the question in the title does have a negative answer:

Let $A=\{a,b,c\}$. Consider the sub-correspondence of the weak Pareto correspondence in which $c$ is removed unless $c$ is the only weak Pareto optimum or $b$ is a weak Pareto optimum. There are two agents. The value under the profile consisting of $a\succ b\succ c$ and $c\succ b\succ a$ is $\{a,b,c\}$. However, under the profile consisting of $a\succ c\succ b$ and $c\succ b\succ a$, the value is $\{a\}$. So this sub-correspondence is not monotonic.

$\endgroup$
1
  • $\begingroup$ Thank you very much for your answer. I have edited the body of the question to ensure it matches the question in the title. $\endgroup$
    – EoDmnFOr3q
    Commented Jul 7 at 7:55

Your Answer

By clicking “Post Your Answer”, you agree to our terms of service and acknowledge you have read our privacy policy.

Not the answer you're looking for? Browse other questions tagged or ask your own question.